square root n limit

The name of the pictureThe name of the pictureThe name of the pictureClash Royale CLAN TAG#URR8PPP












3












$begingroup$


$$(x_n)_ngeq 2 x_n=sqrt[n]1+sum_k=2^n(k-1)(k-1)! $$



$$lim_nrightarrow infty fracx_nn=?$$



I'm trying to solve the sum I but I don't know how to rewrite it.Can you give me some hints?



edit:



The sum $sum_k=2^n(k-1)(k-1)!$ should be $(n! - 1)$ so $x_n=sqrt[n]n!$.The limit would be $lim_nrightarrow infty sqrt[n]fracn!n^n$ .If I use cauchy d'alembert criterion and make some simplifications I would get $e^-1$ (the right answer)My problem is that sum.How I obtain $n! - 1$ ?










share|cite|improve this question











$endgroup$
















    3












    $begingroup$


    $$(x_n)_ngeq 2 x_n=sqrt[n]1+sum_k=2^n(k-1)(k-1)! $$



    $$lim_nrightarrow infty fracx_nn=?$$



    I'm trying to solve the sum I but I don't know how to rewrite it.Can you give me some hints?



    edit:



    The sum $sum_k=2^n(k-1)(k-1)!$ should be $(n! - 1)$ so $x_n=sqrt[n]n!$.The limit would be $lim_nrightarrow infty sqrt[n]fracn!n^n$ .If I use cauchy d'alembert criterion and make some simplifications I would get $e^-1$ (the right answer)My problem is that sum.How I obtain $n! - 1$ ?










    share|cite|improve this question











    $endgroup$














      3












      3








      3





      $begingroup$


      $$(x_n)_ngeq 2 x_n=sqrt[n]1+sum_k=2^n(k-1)(k-1)! $$



      $$lim_nrightarrow infty fracx_nn=?$$



      I'm trying to solve the sum I but I don't know how to rewrite it.Can you give me some hints?



      edit:



      The sum $sum_k=2^n(k-1)(k-1)!$ should be $(n! - 1)$ so $x_n=sqrt[n]n!$.The limit would be $lim_nrightarrow infty sqrt[n]fracn!n^n$ .If I use cauchy d'alembert criterion and make some simplifications I would get $e^-1$ (the right answer)My problem is that sum.How I obtain $n! - 1$ ?










      share|cite|improve this question











      $endgroup$




      $$(x_n)_ngeq 2 x_n=sqrt[n]1+sum_k=2^n(k-1)(k-1)! $$



      $$lim_nrightarrow infty fracx_nn=?$$



      I'm trying to solve the sum I but I don't know how to rewrite it.Can you give me some hints?



      edit:



      The sum $sum_k=2^n(k-1)(k-1)!$ should be $(n! - 1)$ so $x_n=sqrt[n]n!$.The limit would be $lim_nrightarrow infty sqrt[n]fracn!n^n$ .If I use cauchy d'alembert criterion and make some simplifications I would get $e^-1$ (the right answer)My problem is that sum.How I obtain $n! - 1$ ?







      calculus limits






      share|cite|improve this question















      share|cite|improve this question













      share|cite|improve this question




      share|cite|improve this question








      edited Feb 4 at 22:22







      DaniVaja

















      asked Feb 4 at 21:49









      DaniVajaDaniVaja

      977




      977




















          3 Answers
          3






          active

          oldest

          votes


















          5












          $begingroup$

          Very roughly, the sum will be dominated by the last term. You can then apply Stiriling's approximation to the last term
          $$(n-1)(n-1)!approx frac (n-1)^ne^n-1sqrt 2pi (n-1)$$
          The $n^th$ root of this goes to $frac n-1e$ so the limit goes to $frac 1e$
          You need to justify the approximations made, particularly the one about considering only the last term of the sum.






          share|cite|improve this answer









          $endgroup$












          • $begingroup$
            Hi, thank you the response.The sum $sum_k=2^n(k-1)(k-1)!$ should be $n! - 1$ so $x_n=sqrt[n]n!$.The limit would be $lim_nrightarrow infty sqrt[n]fracn!n^n$ .If I use cauchy d'alembert criterion I would get $e^-1$ (the right answer)My problem is that sum.How I obtain $n! - 1$ ?
            $endgroup$
            – DaniVaja
            Feb 4 at 22:18






          • 1




            $begingroup$
            As Vladimir said, you should ask the question you have. I would use induction. Show it is true for $k=2$, which is easy, then assume it is true up to $k$ and show it holds for $k+1$.
            $endgroup$
            – Ross Millikan
            Feb 4 at 23:36


















          2












          $begingroup$

          Well, using Stirling's formula $m!=(m/e)^msqrt2pi m(1+o(1))$ as $mtoinfty$, we find that the radicand for large $k$ is $((n-1)/e)^n-1 A_n$, where $1le A_nle Cn^5/2$ with some positive constant $C$ independent of $n$. Hence $lim_ntoinftysqrt[n]A_n=1$, and we obtain
          $$
          lim_ntoinfty fracx_nn=lim_ntoinftyfrac(n-1)^(n-1)/nne^(n-1)/n=frac1e.
          $$






          share|cite|improve this answer









          $endgroup$












          • $begingroup$
            Hi, thank you the response.The sum $sum_k=2^n(k-1)(k-1)!$ should be $n! - 1$ so $x_n=sqrt[n]n!$.The limit would be $lim_nrightarrow infty sqrt[n]fracn!n^n$ .If I use cauchy d'alembert criterion I would get $e^-1$ (the right answer)My problem is that sum.How I obtain $n! - 1$ ?
            $endgroup$
            – DaniVaja
            Feb 4 at 22:18






          • 1




            $begingroup$
            If that's your real problem, you should have written it in your question in the first place. We only try to answer questions posed and cannot generally read between the lines. Sorry, but I cannot be helpful further with your problem/
            $endgroup$
            – Vladimir
            Feb 4 at 22:22


















          1












          $begingroup$

          Note that $(k-1)(k-1)!$ is very nearly $k(k-1)!$, which is $k!$. We'll use that:
          $$
          sum_k=2^n (k-1)(k-1)! + sum_k=2^n (k-1)! = sum_k=2^n (k-1+1)(k-1)! = sum_k=2^n k!
          $$

          Then (careful with the limits):
          $$
          sum_k=2^n (k-1)(k-1)! = sum_k=2^n k! - sum_k=2^n (k-1)! = sum_k=2^n k! - sum_k=1^n-1 k! = n! - 1! tag*
          $$

          because almost all the terms cancel out.




          Edit: neater train of thought, but technically the same:

          Observe that $(k-1)(k-1)! = k! - (k-1)!$, and then follow with $(*)$.






          share|cite|improve this answer











          $endgroup$












          • $begingroup$
            Thanks a lot! :)
            $endgroup$
            – DaniVaja
            Feb 5 at 21:02










          Your Answer





          StackExchange.ifUsing("editor", function ()
          return StackExchange.using("mathjaxEditing", function ()
          StackExchange.MarkdownEditor.creationCallbacks.add(function (editor, postfix)
          StackExchange.mathjaxEditing.prepareWmdForMathJax(editor, postfix, [["$", "$"], ["\\(","\\)"]]);
          );
          );
          , "mathjax-editing");

          StackExchange.ready(function()
          var channelOptions =
          tags: "".split(" "),
          id: "69"
          ;
          initTagRenderer("".split(" "), "".split(" "), channelOptions);

          StackExchange.using("externalEditor", function()
          // Have to fire editor after snippets, if snippets enabled
          if (StackExchange.settings.snippets.snippetsEnabled)
          StackExchange.using("snippets", function()
          createEditor();
          );

          else
          createEditor();

          );

          function createEditor()
          StackExchange.prepareEditor(
          heartbeatType: 'answer',
          autoActivateHeartbeat: false,
          convertImagesToLinks: true,
          noModals: true,
          showLowRepImageUploadWarning: true,
          reputationToPostImages: 10,
          bindNavPrevention: true,
          postfix: "",
          imageUploader:
          brandingHtml: "Powered by u003ca class="icon-imgur-white" href="https://imgur.com/"u003eu003c/au003e",
          contentPolicyHtml: "User contributions licensed under u003ca href="https://creativecommons.org/licenses/by-sa/3.0/"u003ecc by-sa 3.0 with attribution requiredu003c/au003e u003ca href="https://stackoverflow.com/legal/content-policy"u003e(content policy)u003c/au003e",
          allowUrls: true
          ,
          noCode: true, onDemand: true,
          discardSelector: ".discard-answer"
          ,immediatelyShowMarkdownHelp:true
          );



          );













          draft saved

          draft discarded


















          StackExchange.ready(
          function ()
          StackExchange.openid.initPostLogin('.new-post-login', 'https%3a%2f%2fmath.stackexchange.com%2fquestions%2f3100468%2fsquare-root-n-limit%23new-answer', 'question_page');

          );

          Post as a guest















          Required, but never shown

























          3 Answers
          3






          active

          oldest

          votes








          3 Answers
          3






          active

          oldest

          votes









          active

          oldest

          votes






          active

          oldest

          votes









          5












          $begingroup$

          Very roughly, the sum will be dominated by the last term. You can then apply Stiriling's approximation to the last term
          $$(n-1)(n-1)!approx frac (n-1)^ne^n-1sqrt 2pi (n-1)$$
          The $n^th$ root of this goes to $frac n-1e$ so the limit goes to $frac 1e$
          You need to justify the approximations made, particularly the one about considering only the last term of the sum.






          share|cite|improve this answer









          $endgroup$












          • $begingroup$
            Hi, thank you the response.The sum $sum_k=2^n(k-1)(k-1)!$ should be $n! - 1$ so $x_n=sqrt[n]n!$.The limit would be $lim_nrightarrow infty sqrt[n]fracn!n^n$ .If I use cauchy d'alembert criterion I would get $e^-1$ (the right answer)My problem is that sum.How I obtain $n! - 1$ ?
            $endgroup$
            – DaniVaja
            Feb 4 at 22:18






          • 1




            $begingroup$
            As Vladimir said, you should ask the question you have. I would use induction. Show it is true for $k=2$, which is easy, then assume it is true up to $k$ and show it holds for $k+1$.
            $endgroup$
            – Ross Millikan
            Feb 4 at 23:36















          5












          $begingroup$

          Very roughly, the sum will be dominated by the last term. You can then apply Stiriling's approximation to the last term
          $$(n-1)(n-1)!approx frac (n-1)^ne^n-1sqrt 2pi (n-1)$$
          The $n^th$ root of this goes to $frac n-1e$ so the limit goes to $frac 1e$
          You need to justify the approximations made, particularly the one about considering only the last term of the sum.






          share|cite|improve this answer









          $endgroup$












          • $begingroup$
            Hi, thank you the response.The sum $sum_k=2^n(k-1)(k-1)!$ should be $n! - 1$ so $x_n=sqrt[n]n!$.The limit would be $lim_nrightarrow infty sqrt[n]fracn!n^n$ .If I use cauchy d'alembert criterion I would get $e^-1$ (the right answer)My problem is that sum.How I obtain $n! - 1$ ?
            $endgroup$
            – DaniVaja
            Feb 4 at 22:18






          • 1




            $begingroup$
            As Vladimir said, you should ask the question you have. I would use induction. Show it is true for $k=2$, which is easy, then assume it is true up to $k$ and show it holds for $k+1$.
            $endgroup$
            – Ross Millikan
            Feb 4 at 23:36













          5












          5








          5





          $begingroup$

          Very roughly, the sum will be dominated by the last term. You can then apply Stiriling's approximation to the last term
          $$(n-1)(n-1)!approx frac (n-1)^ne^n-1sqrt 2pi (n-1)$$
          The $n^th$ root of this goes to $frac n-1e$ so the limit goes to $frac 1e$
          You need to justify the approximations made, particularly the one about considering only the last term of the sum.






          share|cite|improve this answer









          $endgroup$



          Very roughly, the sum will be dominated by the last term. You can then apply Stiriling's approximation to the last term
          $$(n-1)(n-1)!approx frac (n-1)^ne^n-1sqrt 2pi (n-1)$$
          The $n^th$ root of this goes to $frac n-1e$ so the limit goes to $frac 1e$
          You need to justify the approximations made, particularly the one about considering only the last term of the sum.







          share|cite|improve this answer












          share|cite|improve this answer



          share|cite|improve this answer










          answered Feb 4 at 22:00









          Ross MillikanRoss Millikan

          297k23198371




          297k23198371











          • $begingroup$
            Hi, thank you the response.The sum $sum_k=2^n(k-1)(k-1)!$ should be $n! - 1$ so $x_n=sqrt[n]n!$.The limit would be $lim_nrightarrow infty sqrt[n]fracn!n^n$ .If I use cauchy d'alembert criterion I would get $e^-1$ (the right answer)My problem is that sum.How I obtain $n! - 1$ ?
            $endgroup$
            – DaniVaja
            Feb 4 at 22:18






          • 1




            $begingroup$
            As Vladimir said, you should ask the question you have. I would use induction. Show it is true for $k=2$, which is easy, then assume it is true up to $k$ and show it holds for $k+1$.
            $endgroup$
            – Ross Millikan
            Feb 4 at 23:36
















          • $begingroup$
            Hi, thank you the response.The sum $sum_k=2^n(k-1)(k-1)!$ should be $n! - 1$ so $x_n=sqrt[n]n!$.The limit would be $lim_nrightarrow infty sqrt[n]fracn!n^n$ .If I use cauchy d'alembert criterion I would get $e^-1$ (the right answer)My problem is that sum.How I obtain $n! - 1$ ?
            $endgroup$
            – DaniVaja
            Feb 4 at 22:18






          • 1




            $begingroup$
            As Vladimir said, you should ask the question you have. I would use induction. Show it is true for $k=2$, which is easy, then assume it is true up to $k$ and show it holds for $k+1$.
            $endgroup$
            – Ross Millikan
            Feb 4 at 23:36















          $begingroup$
          Hi, thank you the response.The sum $sum_k=2^n(k-1)(k-1)!$ should be $n! - 1$ so $x_n=sqrt[n]n!$.The limit would be $lim_nrightarrow infty sqrt[n]fracn!n^n$ .If I use cauchy d'alembert criterion I would get $e^-1$ (the right answer)My problem is that sum.How I obtain $n! - 1$ ?
          $endgroup$
          – DaniVaja
          Feb 4 at 22:18




          $begingroup$
          Hi, thank you the response.The sum $sum_k=2^n(k-1)(k-1)!$ should be $n! - 1$ so $x_n=sqrt[n]n!$.The limit would be $lim_nrightarrow infty sqrt[n]fracn!n^n$ .If I use cauchy d'alembert criterion I would get $e^-1$ (the right answer)My problem is that sum.How I obtain $n! - 1$ ?
          $endgroup$
          – DaniVaja
          Feb 4 at 22:18




          1




          1




          $begingroup$
          As Vladimir said, you should ask the question you have. I would use induction. Show it is true for $k=2$, which is easy, then assume it is true up to $k$ and show it holds for $k+1$.
          $endgroup$
          – Ross Millikan
          Feb 4 at 23:36




          $begingroup$
          As Vladimir said, you should ask the question you have. I would use induction. Show it is true for $k=2$, which is easy, then assume it is true up to $k$ and show it holds for $k+1$.
          $endgroup$
          – Ross Millikan
          Feb 4 at 23:36











          2












          $begingroup$

          Well, using Stirling's formula $m!=(m/e)^msqrt2pi m(1+o(1))$ as $mtoinfty$, we find that the radicand for large $k$ is $((n-1)/e)^n-1 A_n$, where $1le A_nle Cn^5/2$ with some positive constant $C$ independent of $n$. Hence $lim_ntoinftysqrt[n]A_n=1$, and we obtain
          $$
          lim_ntoinfty fracx_nn=lim_ntoinftyfrac(n-1)^(n-1)/nne^(n-1)/n=frac1e.
          $$






          share|cite|improve this answer









          $endgroup$












          • $begingroup$
            Hi, thank you the response.The sum $sum_k=2^n(k-1)(k-1)!$ should be $n! - 1$ so $x_n=sqrt[n]n!$.The limit would be $lim_nrightarrow infty sqrt[n]fracn!n^n$ .If I use cauchy d'alembert criterion I would get $e^-1$ (the right answer)My problem is that sum.How I obtain $n! - 1$ ?
            $endgroup$
            – DaniVaja
            Feb 4 at 22:18






          • 1




            $begingroup$
            If that's your real problem, you should have written it in your question in the first place. We only try to answer questions posed and cannot generally read between the lines. Sorry, but I cannot be helpful further with your problem/
            $endgroup$
            – Vladimir
            Feb 4 at 22:22















          2












          $begingroup$

          Well, using Stirling's formula $m!=(m/e)^msqrt2pi m(1+o(1))$ as $mtoinfty$, we find that the radicand for large $k$ is $((n-1)/e)^n-1 A_n$, where $1le A_nle Cn^5/2$ with some positive constant $C$ independent of $n$. Hence $lim_ntoinftysqrt[n]A_n=1$, and we obtain
          $$
          lim_ntoinfty fracx_nn=lim_ntoinftyfrac(n-1)^(n-1)/nne^(n-1)/n=frac1e.
          $$






          share|cite|improve this answer









          $endgroup$












          • $begingroup$
            Hi, thank you the response.The sum $sum_k=2^n(k-1)(k-1)!$ should be $n! - 1$ so $x_n=sqrt[n]n!$.The limit would be $lim_nrightarrow infty sqrt[n]fracn!n^n$ .If I use cauchy d'alembert criterion I would get $e^-1$ (the right answer)My problem is that sum.How I obtain $n! - 1$ ?
            $endgroup$
            – DaniVaja
            Feb 4 at 22:18






          • 1




            $begingroup$
            If that's your real problem, you should have written it in your question in the first place. We only try to answer questions posed and cannot generally read between the lines. Sorry, but I cannot be helpful further with your problem/
            $endgroup$
            – Vladimir
            Feb 4 at 22:22













          2












          2








          2





          $begingroup$

          Well, using Stirling's formula $m!=(m/e)^msqrt2pi m(1+o(1))$ as $mtoinfty$, we find that the radicand for large $k$ is $((n-1)/e)^n-1 A_n$, where $1le A_nle Cn^5/2$ with some positive constant $C$ independent of $n$. Hence $lim_ntoinftysqrt[n]A_n=1$, and we obtain
          $$
          lim_ntoinfty fracx_nn=lim_ntoinftyfrac(n-1)^(n-1)/nne^(n-1)/n=frac1e.
          $$






          share|cite|improve this answer









          $endgroup$



          Well, using Stirling's formula $m!=(m/e)^msqrt2pi m(1+o(1))$ as $mtoinfty$, we find that the radicand for large $k$ is $((n-1)/e)^n-1 A_n$, where $1le A_nle Cn^5/2$ with some positive constant $C$ independent of $n$. Hence $lim_ntoinftysqrt[n]A_n=1$, and we obtain
          $$
          lim_ntoinfty fracx_nn=lim_ntoinftyfrac(n-1)^(n-1)/nne^(n-1)/n=frac1e.
          $$







          share|cite|improve this answer












          share|cite|improve this answer



          share|cite|improve this answer










          answered Feb 4 at 22:04









          VladimirVladimir

          5,393618




          5,393618











          • $begingroup$
            Hi, thank you the response.The sum $sum_k=2^n(k-1)(k-1)!$ should be $n! - 1$ so $x_n=sqrt[n]n!$.The limit would be $lim_nrightarrow infty sqrt[n]fracn!n^n$ .If I use cauchy d'alembert criterion I would get $e^-1$ (the right answer)My problem is that sum.How I obtain $n! - 1$ ?
            $endgroup$
            – DaniVaja
            Feb 4 at 22:18






          • 1




            $begingroup$
            If that's your real problem, you should have written it in your question in the first place. We only try to answer questions posed and cannot generally read between the lines. Sorry, but I cannot be helpful further with your problem/
            $endgroup$
            – Vladimir
            Feb 4 at 22:22
















          • $begingroup$
            Hi, thank you the response.The sum $sum_k=2^n(k-1)(k-1)!$ should be $n! - 1$ so $x_n=sqrt[n]n!$.The limit would be $lim_nrightarrow infty sqrt[n]fracn!n^n$ .If I use cauchy d'alembert criterion I would get $e^-1$ (the right answer)My problem is that sum.How I obtain $n! - 1$ ?
            $endgroup$
            – DaniVaja
            Feb 4 at 22:18






          • 1




            $begingroup$
            If that's your real problem, you should have written it in your question in the first place. We only try to answer questions posed and cannot generally read between the lines. Sorry, but I cannot be helpful further with your problem/
            $endgroup$
            – Vladimir
            Feb 4 at 22:22















          $begingroup$
          Hi, thank you the response.The sum $sum_k=2^n(k-1)(k-1)!$ should be $n! - 1$ so $x_n=sqrt[n]n!$.The limit would be $lim_nrightarrow infty sqrt[n]fracn!n^n$ .If I use cauchy d'alembert criterion I would get $e^-1$ (the right answer)My problem is that sum.How I obtain $n! - 1$ ?
          $endgroup$
          – DaniVaja
          Feb 4 at 22:18




          $begingroup$
          Hi, thank you the response.The sum $sum_k=2^n(k-1)(k-1)!$ should be $n! - 1$ so $x_n=sqrt[n]n!$.The limit would be $lim_nrightarrow infty sqrt[n]fracn!n^n$ .If I use cauchy d'alembert criterion I would get $e^-1$ (the right answer)My problem is that sum.How I obtain $n! - 1$ ?
          $endgroup$
          – DaniVaja
          Feb 4 at 22:18




          1




          1




          $begingroup$
          If that's your real problem, you should have written it in your question in the first place. We only try to answer questions posed and cannot generally read between the lines. Sorry, but I cannot be helpful further with your problem/
          $endgroup$
          – Vladimir
          Feb 4 at 22:22




          $begingroup$
          If that's your real problem, you should have written it in your question in the first place. We only try to answer questions posed and cannot generally read between the lines. Sorry, but I cannot be helpful further with your problem/
          $endgroup$
          – Vladimir
          Feb 4 at 22:22











          1












          $begingroup$

          Note that $(k-1)(k-1)!$ is very nearly $k(k-1)!$, which is $k!$. We'll use that:
          $$
          sum_k=2^n (k-1)(k-1)! + sum_k=2^n (k-1)! = sum_k=2^n (k-1+1)(k-1)! = sum_k=2^n k!
          $$

          Then (careful with the limits):
          $$
          sum_k=2^n (k-1)(k-1)! = sum_k=2^n k! - sum_k=2^n (k-1)! = sum_k=2^n k! - sum_k=1^n-1 k! = n! - 1! tag*
          $$

          because almost all the terms cancel out.




          Edit: neater train of thought, but technically the same:

          Observe that $(k-1)(k-1)! = k! - (k-1)!$, and then follow with $(*)$.






          share|cite|improve this answer











          $endgroup$












          • $begingroup$
            Thanks a lot! :)
            $endgroup$
            – DaniVaja
            Feb 5 at 21:02















          1












          $begingroup$

          Note that $(k-1)(k-1)!$ is very nearly $k(k-1)!$, which is $k!$. We'll use that:
          $$
          sum_k=2^n (k-1)(k-1)! + sum_k=2^n (k-1)! = sum_k=2^n (k-1+1)(k-1)! = sum_k=2^n k!
          $$

          Then (careful with the limits):
          $$
          sum_k=2^n (k-1)(k-1)! = sum_k=2^n k! - sum_k=2^n (k-1)! = sum_k=2^n k! - sum_k=1^n-1 k! = n! - 1! tag*
          $$

          because almost all the terms cancel out.




          Edit: neater train of thought, but technically the same:

          Observe that $(k-1)(k-1)! = k! - (k-1)!$, and then follow with $(*)$.






          share|cite|improve this answer











          $endgroup$












          • $begingroup$
            Thanks a lot! :)
            $endgroup$
            – DaniVaja
            Feb 5 at 21:02













          1












          1








          1





          $begingroup$

          Note that $(k-1)(k-1)!$ is very nearly $k(k-1)!$, which is $k!$. We'll use that:
          $$
          sum_k=2^n (k-1)(k-1)! + sum_k=2^n (k-1)! = sum_k=2^n (k-1+1)(k-1)! = sum_k=2^n k!
          $$

          Then (careful with the limits):
          $$
          sum_k=2^n (k-1)(k-1)! = sum_k=2^n k! - sum_k=2^n (k-1)! = sum_k=2^n k! - sum_k=1^n-1 k! = n! - 1! tag*
          $$

          because almost all the terms cancel out.




          Edit: neater train of thought, but technically the same:

          Observe that $(k-1)(k-1)! = k! - (k-1)!$, and then follow with $(*)$.






          share|cite|improve this answer











          $endgroup$



          Note that $(k-1)(k-1)!$ is very nearly $k(k-1)!$, which is $k!$. We'll use that:
          $$
          sum_k=2^n (k-1)(k-1)! + sum_k=2^n (k-1)! = sum_k=2^n (k-1+1)(k-1)! = sum_k=2^n k!
          $$

          Then (careful with the limits):
          $$
          sum_k=2^n (k-1)(k-1)! = sum_k=2^n k! - sum_k=2^n (k-1)! = sum_k=2^n k! - sum_k=1^n-1 k! = n! - 1! tag*
          $$

          because almost all the terms cancel out.




          Edit: neater train of thought, but technically the same:

          Observe that $(k-1)(k-1)! = k! - (k-1)!$, and then follow with $(*)$.







          share|cite|improve this answer














          share|cite|improve this answer



          share|cite|improve this answer








          edited Feb 5 at 0:07

























          answered Feb 4 at 23:55









          eudeseudes

          1054




          1054











          • $begingroup$
            Thanks a lot! :)
            $endgroup$
            – DaniVaja
            Feb 5 at 21:02
















          • $begingroup$
            Thanks a lot! :)
            $endgroup$
            – DaniVaja
            Feb 5 at 21:02















          $begingroup$
          Thanks a lot! :)
          $endgroup$
          – DaniVaja
          Feb 5 at 21:02




          $begingroup$
          Thanks a lot! :)
          $endgroup$
          – DaniVaja
          Feb 5 at 21:02

















          draft saved

          draft discarded
















































          Thanks for contributing an answer to Mathematics Stack Exchange!


          • Please be sure to answer the question. Provide details and share your research!

          But avoid


          • Asking for help, clarification, or responding to other answers.

          • Making statements based on opinion; back them up with references or personal experience.

          Use MathJax to format equations. MathJax reference.


          To learn more, see our tips on writing great answers.




          draft saved


          draft discarded














          StackExchange.ready(
          function ()
          StackExchange.openid.initPostLogin('.new-post-login', 'https%3a%2f%2fmath.stackexchange.com%2fquestions%2f3100468%2fsquare-root-n-limit%23new-answer', 'question_page');

          );

          Post as a guest















          Required, but never shown





















































          Required, but never shown














          Required, but never shown












          Required, but never shown







          Required, but never shown

































          Required, but never shown














          Required, but never shown












          Required, but never shown







          Required, but never shown






          Popular posts from this blog

          How to check contact read email or not when send email to Individual?

          Bahrain

          Postfix configuration issue with fips on centos 7; mailgun relay